Soal dan Pembahasan – ON MIPA-PT Matematika Bidang Analisis Real

Soal ON MIPA analisis real

Berikut ini adalah soal-soal ON MIPA-PT Bidang Analisis Real beserta pembahasannya. Semoga bermanfaat dan salam sukses, pejuang ON MIPA! 

Baca Juga: Soal dan Pembahasan- ON MIPA-PT Bidang Struktur Aljabar

Baca Juga: Soal dan Pembahasan- ON MIPA-PT Bidang Analisis Kompleks

Baca Juga: Soal dan Pembahasan- ON MIPA-PT Bidang Aljabar Linear

Quote by Rod Stewart

You go through life wondering what is it all about, but at the end of the day it’s all about family.

Soal Nomor 1

Diberikan himpunan $A = \{3^{2x} + 3^{\frac{1}{2x}} | x > 0\}$. Tentukan infimum $A$.

Pembahasan

Dengan menggunakan ketaksamaan Aritmetik-Geometri, diperoleh
$\begin{aligned} 3^{2x} + 3^{\frac{1}{2x}} &\geq 2\left(3^{2x}.3^{\frac{1}{2x}}\right)^{\frac{1}{2}} \\ & = 2\left(3^{2x + (2x)^{-1}} \right)^{\frac{1}{2}} \\ & \geq 2.3^{\left((2x)(2x) ^{-1}\right)\frac{1}{2}} \\ & = 2 \times 3 = 6 \end{aligned}$
dan persamaannya berlaku jika dan hanya jika $3^{2x} = 3^{\frac{1}{2x}}$ dan $2x = (2x) ^{-1}.$ Untuk $x > 0$, kita bisa mengambil $x = \dfrac{1}{2}$ untuk memenuhi kondisi tersebut. Jadi, infimum $A$ adalah $\boxed{6}$

[collapse]

Soal Nomor 2

Jika barisan bilangan real $(x_n)$ memenuhi sifat $\displaystyle \lim_{n \to \infty} x_{2n} + x_{2n+1}= 315$ dan $\displaystyle \lim_{n \to \infty} x_{2n} + x_{2n-1}= 2016$, maka $\displaystyle \lim_{n \to \infty} \dfrac{x_{2n}}{x_{2n+1}} = \cdots \cdot$

Pembahasan

Dengan menggunakan Teorema Stolz-Cesaro, diperoleh
$$\begin{aligned} \displaystyle \lim_{n \to \infty} \dfrac{x_{2n}}{x_{2n+1}} & =\lim_{n \to \infty} \dfrac{x_{2n}- x_{2(n-1)}}{x_{2n+1}- x_{2(n-1)+1}} \\ & = \lim_{n \to \infty} \dfrac{x_{2n}- x_{2n-2}}{x_{2n+1}- x_{2n- 1}} \\ &= \lim_{n \to \infty} \dfrac{x_{2n} + x_{2n-1}- (x_{2n-2} + x_{2n-1})}{x_{2n+1} + x_{2n}- (x_{2n} + x_{2n-1})} \\ & = \dfrac{2016-315}{315-2016} =-1. \end{aligned}$$Jadi, didapat $\boxed{\displaystyle \lim_{n \to \infty} \dfrac{x_{2n}}{x_{2n+1}} =-1}$

[collapse]

Soal Nomor 3

Diketahui $a < \dfrac{\pi} {2}$. Jika $M < 1$ dengan $|\cos x- \cos y| \leq M|x-y|$ untuk setiap $x, y \in [0,a]$, maka $M = \cdots \cdot$

Pembahasan

Perhatikan bahwa jika fungsi $f$ terdiferensialkan pada interval $I$, maka
$$f~\text{fungsi lipschitz}~\Leftrightarrow f'(x)~\text{terbatas di}~I$$dan
$M = \sup\{|f'(x)|, x \in I\}.$
Dalam kasus ini, kita mendapatkan $f(x) = \cos x$ sehingga $f'(x) =-\sin x$. Dengan demikian,

$\begin{aligned} M & = \sup\{|-\sin x|, x \in [0,a]\} \\ & = \sup\{\sin x,  x \in [0,a]\} \\ & = \sin a \end{aligned}$
Jadi, $\boxed{M = \sin a}$

[collapse]

Soal Nomor 4

Diketahui fungsi
$f(x) = \begin{cases} \sin 2x, & x \leq 0 \\ ax, & 0 < x < 1 \\ x^2+b, & x \geq 1 \end{cases}$
mempunyai turunan di $x = 0$ dan $x = 1.$
Tentukan nilai dari $a-b$.

Pembahasan

$f(x)$ memiliki turunan di $x = 0$ dan $x = 1,$ berarti fungsi itu kontinu di titik-titik tersebut.
Perhatikan bahwa
$\begin{aligned} \displaystyle & \lim_{x \to 1^{-}} f(x) = f(1) \\ & \lim_{x \to 1} ax = f(1) \\ & a = (1)^2 + b \\ & \boxed{a- b = 1} \end{aligned} $
Catatan:
Untuk memeriksa masing-masing nilai $a$ dan $b$, diferensialkan fungsinya,
$f'(x) = \begin{cases} 2 \cos 2x, & x \leq 0 \\ a, & 0 < x < 1 \\ 2x, & x \geq 1 \end{cases}$
Agar fungsinya kontinu, haruslah $2 \cos 2(0) = a$, dan mengimplikasikan $a = 2$ dan $b = 1$.

[collapse]

Soal Nomor 5

Diberikan deret pangkat $\displaystyle \sum_{k=0}^{\infty} \dfrac{x^k} {k^2+1}$.
Tentukan himpunan/selang kekonvergenan deret itu. 

Pembahasan

Bentuk sumasinya dapat diubah dalam bentuk deret pangkat, yaitu
$\displaystyle \sum_{k=0}^{\infty} \dfrac{x^k} {k^2+1} = \sum_{k=0}^{\infty} C_k(x-0)^k.$
Dengan menggunakan uji rasio, diperoleh
$$\begin{aligned} \displaystyle \lim_{k \to \infty} \dfrac{|C_{k+1}(x)|} {|C_k(x)|} & = \lim_{k \to \infty} \dfrac{\left|\dfrac{1}{(k+1)^2+1}\right|\left|x^{k+1}\right|} {\left|\dfrac{1}{k^2+1}\right| \left|x^k\right|} \\ & = |x| \lim_{k \to \infty} \left|\dfrac{k^2+1}{k^2+2k+2}\right| \\ & = |x|. \end{aligned}$$Berdasarkan teorema uji rasio, deret itu akan konvergen jika $|x| < 1.$ Berikutnya, uji konvergensi deret di ujung titik. Ketika $x = -1$ atau $x = 1,$ deret tersebut konvergen. Akibatnya, selang kekonvergenan deret tersebut adalah $\boxed{[-1, 1]}$

[collapse]

Soal Nomor 6

Tentukan nilai dari $\displaystyle \sum_{k=1}^{\infty} \dfrac{(-x)^{k+1}} {k} $ dan jari-jari konvergensinya.

Pembahasan

Perhatikan bahwa
$$\begin{aligned} \displaystyle \sum_{k=1}^{\infty} \dfrac{(-x)^{k+1}} {k} & = x^2- \dfrac{1}{2}x^3 + \dfrac{1}{3}x^4- \cdots + (-1)^{n+1}\dfrac{x^{n+1}} {n} + \cdots \\ & = x\left(x- \dfrac{1}{2}x^2 + \dfrac{1}{3}x^3- \cdots + (-1)^{n+1}\dfrac{x^{n}} {n} + \cdots \right) \\ & = x(\ln |1 + x|). \end{aligned}$$dengan jari-jari konvergensinya $(-1, 1).$

Catatan:
Bentuk khusus (saat $x = 1$)
$\displaystyle \sum_{k=1}^{\infty} \dfrac{(-1)^{k+1}} {k} = \ln 2$
sering muncul dalam beberapa kasus.

[collapse]

Soal Nomor 7

Diberikan fungsi tak nol $f: D \mapsto \mathbb{R}$ dan fungsi $g: D \mapsto \mathbb{R} $ dengan $D \subseteq \mathbb{R}$ sedemikian sehingga $\dfrac{f(x)} {g(x)} \leq 1, \forall x \in D.$
Berilah contoh fungsi $f$ dan $g$ yang menunjukkan bahwa belum tentu berlaku $\displaystyle \sup_{x \in D} g(x) \leq \inf_{x \in D} f(x).$

Pembahasan

Diketahui untuk setiap $x \in \mathbb{R}$, berlaku
$\begin{aligned} & x^2 \geq 0 \\ & 2x^2 \ge x^2 \geq 0 \\ & 1 + 2x^2 \geq 1 + x^2 > 0 \\ & \dfrac{1}{1+2x^2} \leq \dfrac{1}{1+x^2} \\ & \dfrac{1+x^2}{1+2x^2} \leq 1. \end{aligned}$
Ambil $g(x) = \dfrac{1}{1+2x^2}$ dan $f(x) = \dfrac{1}{1+x^2}$ sehingga terpenuhi
$\dfrac{g(x)} {f(x)} \leq 1.$
Ini menunjukkan bahwa
$\displaystyle \sup_{x \in D} g(x) = 1 \leq \inf_{x \in D} f(x) = 0$.

[collapse]

Soal Nomor 8

Tentukan nilai dari $\displaystyle \lim_{k \to \infty} \dfrac{1}{k^k} \sum_{n=1}^{k} n^k$.

Pembahasan

Perhatikan bahwa bentuk di atas dapat ditulis menjadi
$$\begin{aligned} & \displaystyle \lim_{k \to \infty} \sum_{n=1}^{k} \left(\dfrac{n} {k}\right)^k \\ & = \lim_{k \to \infty} \left(\left(\dfrac{1}{k}\right)^k +\left(\dfrac{2}{k}\right)^k + \cdots + \left(\dfrac{k}{k}\right)^k\right) \\ & = \lim_{k \to \infty} \left(1 + \left(1 + \dfrac{-1}{k}\right)^k + \left(1 + \dfrac{-2}{k}\right)^k + \cdots + \left(1 + \dfrac{-k}{k}\right)^k\right) \\ & = 1 + e^{-1} + e^{-2} + e^{-3} + \cdots \\ & = \dfrac{1}{1-e^{-1}} = \boxed{\dfrac{e} {e- 1}} \end{aligned} $$Catatan:
Ingat bahwa $\displaystyle \lim_{n \to \infty} \left(1 + \dfrac{x} {n} \right)^n = e^x$ untuk setiap $x \in \mathbb{R}$.

[collapse]

Soal Nomor 9

Jika $f$ fungsi kontinu pada selang $[0, \infty]$ dan  $\displaystyle \int_0^{x^2} f(t)~dt = x(\cos (\pi x)- 1).$ Hitung $f(9)$.

Pembahasan

Teorema Dasar Kalkulus Pertama mengatakan bahwa untuk setiap fungsi $f$ yang kontinu pada interval tertutup $[a, b]$ dan $x$ sembarang titik dalam interval tersebut, maka berlaku
$\displaystyle \dfrac{d}{dx} \int_0^x f(t)~dt = f(x).$
Jadi,
$$\begin{aligned} \dfrac{d}{dx} \int_0^{x^2} f(t)~dt & = \dfrac{d}{dx}\left[F(t)\right]_0^{x^2} \\ & = \dfrac{d} {dx} (F(x^2)- F(0)) \\ & = f(x^2). 2x = 2xf(x^2). \end{aligned} $$Selanjutnya, dapat kita tuliskan
$$\begin{aligned} \displaystyle \dfrac{d}{dx} \int_0^{x^2} f(t)~dt & = \dfrac{d}{dx} x(\cos (\pi x)- 1) \\ 2xf(x^2) & = (\cos \pi x- 1)- \pi x \sin \pi x \\ 2 \cdot 3f(3^2) & = \cos 3\pi- 1- 3\pi \sin 3\pi \\ f(9) &=- \dfrac{1}{3}. \end{aligned}$$Jadi, nilai dari $f(9)$ adalah $\boxed{-\dfrac{1}{3}}$
Catatan:
Turunan $x$ terhadap fungsi konstan $f(0) = 0$ adalah $f'(0)=0$.

[collapse]

Soal Nomor 10

Diberikan $\theta_n = \arctan n $, maka
$\displaystyle \lim_{n \to \infty} (\theta_{n+1}- \theta_n) = \cdots \cdot$

Pembahasan

Perhatikan bahwa
$\begin{aligned} \displaystyle & \lim_{n \to \infty} (\theta_{n+1}- \theta_n) \\ & = \lim_{n \to \infty} (\arctan (n+1)- \arctan n) \\ & = \dfrac{\pi} {2}- \dfrac{\pi} {2} = 0. \end{aligned}$
Jadi, diperoleh $\boxed{\displaystyle \lim_{n \to \infty} (\theta_{n+1}- \theta_n) = 0}$ 

[collapse]

Soal Nomor 11

Diberikan fungsi $f: \mathbb{R} \mapsto \mathbb{R}$ dengan
$f(x) = 1 + a_1 \sin x + a_2 \sin 2x + \cdots$ $+ a_n \sin nx$ untuk $\forall x \in \mathbb{R}$. Jika $|f(x)-1| = 2 \sin 2x$, maka nilai $|a_1 + 2a_2 + \cdots + na_n| = \cdots \cdot$

Pembahasan

Perhatikan bahwa
$$\begin{aligned} & f(x) = 1 + a_1 \sin x + a_2 \sin 2x + \cdots + a_n \sin nx \\ & f'(x) = a_1 \cos x + 2a_2 \cos 2x + \cdots + na_n \cos nx \\ & f'(0) = a_1 + 2a_2 + \cdots na_n. \end{aligned} $$Diberikan $|f(x)-1| = 2 \sin 2x$ sehingga
$$f(x)-1 = \begin{cases} 2 \sin 2x, & x \in \left[-\dfrac{\pi}{2}, \dfrac{\pi} {2}\right] \\ \\-2 \sin 2x, & x \in \left[-\pi,-\dfrac{\pi} {2}\right) \cup \left(\dfrac{\pi}{2}, \pi \right] \end{cases} $$Dari sini, kita dapatkan turunannya,$f'(x) = \begin{cases} 4 \cos 2x \\-4 \cos 2x \end{cases}$
untuk nilai $x$ yang bersesuaian sehingga
$f'(0) = \begin{cases} 4 \\-4 \end{cases}$
Akibatnya, $|f'(0)| = 4.$
Jadi, kita peroleh
$\boxed{|a_1 + 2a_2 + \cdots + na_n| = |f'(0)| = 4}$

[collapse]

Soal Nomor 12

Misalkan $a_i > 0, \forall i = 1,2,\cdots, 2016$. Jika $(a_1a_2\cdots a_{2016})^{\frac{1}{2016}} = 2,$ maka $(1 + a_1)(1 + a_2)\cdots(1 + a_{2016}) \geq \cdots \cdot$

Pembahasan

Persamaan $(a_1a_2\cdots a_{2016})^{\frac{1}{2016}} = 2$ mencapai nilai minimum saat $a_1 = a_2 = \cdots = a_{2016} = 2$ sehingga
$\begin{aligned} & (1 + a_1)(1 + a_2)\cdots(1 + a_{2016}) \\ &  \geq (1 + 2)(1+2)\cdots (1+2) = 3^{2016}. \end{aligned}$

[collapse]

Soal Nomor 13

Untuk setiap $n \in \mathbb{N}, f_n = nx(1- x^2)^n$ untuk setiap $x$ dengan $0 \leq x \leq 1$ dan $a_n = \displaystyle \int_0^1 f_n(x)~dx$. Jika $s_n = \sin (\pi a_n)$ untuk setiap $n \in \mathbb{N}$, maka $\displaystyle \lim_{n \to \infty} s_n = \cdots \cdot$

Pembahasan

Diberikan $f_n = nx(1- x^2)^n$
dan
$\begin{aligned} \displaystyle a_n  & = \int_0^1 f_n(x)~\text{d}x \\ & = \int_0^1 nx(1-x^2)^n~\text{d}x . \end{aligned}$
Dengan menggunakan metode substitusi dalam integral, kita misalkan $u = 1- x^2$ sehingga $\text{d}u = -2x~\text{d}x$, yang berarti $\text{d}x = -\dfrac{1}{2x}~\text{d}u$. Dengan demikian, didapat
$\begin{aligned} a_n & = \displaystyle \int_0^1 nx(u)^n \cdot \left(-\dfrac{1}{2x}\right)~\text{d}u \\ & = -\dfrac{n}{2} \int_0^1 u^n~\text{d}{u} \\ & = -\left[\dfrac{n}{2} \cdot \dfrac{1}{n+1}u^{n+1}\right]_0^1 \\ & = \left[-\dfrac{n} {2(n+1)} (1-x^2)^{n+1}\right]_0^1 \\ & = \dfrac{n}{2(n+1)}. \end{aligned}$
Jadi,
$\boxed{\begin{aligned} \displaystyle \lim_{n \to \infty} s_n & = \lim_{n \to \infty} \sin\left(\pi \times \dfrac{n} {2(n+1)}\right) \\ & = \sin \dfrac{\pi} {2}= 1 \end{aligned}}$

[collapse]

Soal Nomor 14

Jika $E = \{f | f: \mathbb{R} \mapsto \mathbb{R}$ fungsi kontinu dengan $f(x) \in \mathbb{Q}, \forall x \in \mathbb{R}\}$, maka $E = \cdots \cdot$

Pembahasan

Diketahui $E = \{f(x) = c, c \in \mathbb{Q}\}$.
Dengan menggunakan kontradiksi, andaikan $f \in E$ dan $f$ tak konstan, maka akan ditemukan dua bilangan real $x$ dan $y$ sedemikian sehingga $f(x) \neq f(y) $. Ambil sebuah bilangan irasional $z$ di antara $f(x)$ dan $f(y)$. Dengan menggunakan Teorema Nilai Antara (Intermediate Value Theorem), ada $w \in \mathbb{R}$ sedemikian sehingga $f(w) = z$ dan ini kontradiktif dengan definisi himpunan $E$ karena $f(w) \in \mathbb{Q}$ harus terpenuhi untuk setiap $w \in \mathbb{R}$. Jadi, haruslah $f$ konstan berupa bilangan rasional.

[collapse]

Soal Nomor 15

Jika $S = \{\sqrt[3]{n}- \sqrt[3]{m}~|~m, n \in \mathbb{N}\}$, maka $\sup (S) = \cdots \cdot$

Pembahasan

Dengan menggunakan pendekatan limit, diperoleh
$\displaystyle \lim_{m \to \infty} (\sqrt[3]{n}- \sqrt[3]{m}) =-\infty$
dan juga
$\displaystyle \lim_{n \to \infty} (\sqrt[3]{n}- \sqrt[3]{m}) = \infty.$
Dengan demikian, supremum dari $S$ tidak ada karena tidak terbatas di atas.

[collapse]

Soal Nomor 16

Infimum dari himpunan $\{n \in \mathbb{N} : (n!)\}$ adalah $\cdots \cdot$

Pembahasan

Didefinisikan $n! = n(n-1)(n-2)\cdots(2)(1).$
Misalkan $A = \{n \in \mathbb{N} : (n!)\}.$
Dengan demikian, infimum dari himpunan tersebut diperoleh saat $n$ minimum ($n = 1$) sehingga $\boxed{\inf (A) = 1}$

[collapse]

Soal Nomor 17

Jika $f$ kontinu pada $\mathbb{R}$ dan $\displaystyle \int_0^{x^2} t f(t)~\text{d}t = x^6-2x^4$, tentukan $f(1)+f(4)$.

Pembahasan

Misalkan $u = x^2$
Menurut Teorema Dasar Kalkulus Pertama (TDKP), berlaku
$\begin{aligned} & \displaystyle \dfrac{\text{d}} {\text{d}u} \int_0^u t f(t)~\text{d}t \cdot \left(\dfrac{\text{d}} {\text{d}x} \cdot x^2\right) \\ & = u \cdot f(u) \cdot 2x \\ & \text{Substitusi kembali}~u=x^2 \\ & x^2 \cdot f(x^2) \cdot 2x = 2x^3f(x^2). \end{aligned}$
Jadi, 
$$\begin{aligned} \displaystyle \int_0^{x^2} t f(t)~\text{d}t & = x^6-2x^4 \\ \text{Turunkan kedua ruas}~&\text{terhadap variabel}~x \\ \dfrac{\text{d}} {\text{d}x} \int_0^{x^2} t f(t)~\text{d}t & = \dfrac{\text{d}} {\text{d}x} (x^6-2x^4) \\ 2x^3f(x^2) & = 6x^5-8x^3 \\ f(x^2) & = 3x^2-4 \\ f(x) & = 3x-4. \end{aligned}$$Dengan demikian, 
$$\boxed{\begin{aligned} f(1)+f(4) & =(3(1)-4) + (3(4)-4) \\ & =-1+8 = 7 \end{aligned}}$$

[collapse]

Soal Nomor 18

Misalkan $x_n = \displaystyle \sum_{k=1}^n \dfrac{2}{2n+2k-1}.$
Hitung nilai dari $\displaystyle \lim_{n \to \infty} x_n$.

Pembahasan

Perhatikan bahwa
$\begin{aligned} x_n & = \displaystyle \sum_{k=1}^n \dfrac{2}{2n+2k-1} \\ & = \dfrac{1}{n} \sum_{k=1}^n \dfrac{2n}{2n+2k-1} \\ & = \sum_{k=1}^n \dfrac{1}{1 + k \cdot \dfrac{1}{n}-\dfrac{1}{2n}} \cdot \dfrac{1}{n}. \end{aligned}$
Untuk itu,
$$\displaystyle \lim_{n \to \infty} x_n = \lim_{n \to \infty} \sum_{k=1}^n \dfrac{1}{1 + k \cdot \color{red}{\dfrac{1}{n}}-\dfrac{1}{2n}} \cdot \color{red}{\dfrac{1}{n}}.$$Bentuk di atas merupakan Jumlah Riemann untuk $n$ menuju tak hingga.
Dari bentuk tersebut, diperoleh $\Delta x = \dfrac{1}{n}$ sehingga $x_1 = \dfrac{1}{n}$, $x_2 =2 \cdot \dfrac{1}{n}$, sampai $x_n = n \cdot \dfrac{1}{n}=1$.
Batas bawah integral tentunya adalah $a = \displaystyle \lim_{n \to \infty} x_1 = \lim_{n \to \infty} \dfrac{1}{n} = 0$, sedangkan batas atas integral tentunya adalah $b = \displaystyle \lim_{n \to \infty} x_n = \lim_{n \to \infty} 1= 1$.
Dengan demikian, integral tentu yang dinyatakan oleh Jumlah Riemann tersebut adalah $\displaystyle \int_a^b f(x)~\text{d}x = \int_0^1 \dfrac{1}{1+x}~\text{d}x$.
Selanjutnya, didapat

$$\begin{aligned} \displaystyle  \int_0^1 \dfrac{1}{1+x}~\text{d}x & = \left[\ln (1+x)\right]_0^1 \\ & = \ln (1+1)- \ln (1+0) \\ & = \ln 2- \ln 1 = \ln 2. \end{aligned}$$Jadi, nilai dari $\boxed{\displaystyle \lim_{n \to \infty} x_n = \ln 2}$

[collapse]

Soal Nomor 19

Nyatakan limit berikut dalam integral tentu. 
$\displaystyle \lim_{n \to \infty} \dfrac{\pi} {2n} \sum_{k=1}^n \cos \left(\dfrac{k\pi} {2n} \right)$

Pembahasan

Bentuk limit di atas dapat diubah menjadi
$\displaystyle \lim_{n \to \infty} \color{red}{\dfrac{\dfrac{\pi} {2}} {n}} \sum_{k=1}^n \cos \left(k \cdot \left(\color{red}{\dfrac{\dfrac{\pi}{2}} {n}} \right)\right).$
Dari bentuk tersebut, diperoleh $\Delta x = \dfrac{\dfrac{\pi}{2}}{n}$ sehingga $x_1 = \dfrac{\dfrac{\pi}{2}}{n}$, $x_2 =2 \cdot \dfrac{\dfrac{\pi}{2}}{n}$, sampai $x_n = n \cdot \dfrac{\dfrac{\pi}{2}}{n}=\dfrac{\pi}{2}$.
Batas bawah integral tentunya adalah $a = \displaystyle \lim_{n \to \infty} x_1 = \lim_{n \to \infty} \dfrac{\dfrac{\pi}{2}}{n} = 0$, sedangkan batas atas integral tentunya adalah $b = \displaystyle \lim_{n \to \infty} x_n = \lim_{n \to \infty} \dfrac{\pi}{2}= \dfrac{\pi}{2}$.
Dengan demikian, integral tentu yang dinyatakan oleh Jumlah Riemann tersebut adalah $\displaystyle \int_a^b f(x)~\text{d}x = \int_0^{\pi/2} \cos x~\text{d}x$. 

[collapse]

Baca Juga: Soal dan Pembahasan – Jumlah Riemann

Soal Nomor 20

Diberikan fungsi $f: \mathbb{R} \mapsto \mathbb{R}$ dengan $f(x) = 4x^2 + 1$ untuk setiap $x \in \mathbb{R}$ dan barisan $x_n = \displaystyle \sum_{k=1}^n \dfrac{1}{k^2+5k+6}$ untuk setiap $n \in \mathbb{N}$. Nilai $\displaystyle \lim_{x \to \infty} f(x_n) = \cdots \cdot$

Pembahasan

Tinjau bentuk $x_n = \displaystyle \sum_{k=1}^n \dfrac{1}{k^2+5k+6}.$
$$\begin{aligned} \displaystyle \sum_{k=1}^n \dfrac{1}{k^2+5k+6}  & = \sum_{k=1}^n \dfrac{1}{(k+2)(k+3)} \\ & = \sum_{k=1}^n \left(\dfrac{1}{k+2}- \dfrac{1}{k+3}\right) \end{aligned}$$Untuk $n \to \infty$, kita peroleh bentuk penguraian sigmanya menjadi
$$\begin{aligned} & \left(\dfrac{1}{1+2}- \cancel{\dfrac{1}{1+3}}\right) + \left(\cancel{\dfrac{1}{2+2}}- \cancel{\dfrac{1}{2+3}} \right) \\ & + \left(\cancel{\dfrac{1}{3+2}}- \cancel{\dfrac{1}{3+3}}\right) + \cdots = \dfrac{1}{3} \end{aligned}$$Untuk $x = \dfrac{1}{3}$, diperoleh
$f\left(\dfrac{1}{3}\right) = 4\left(\dfrac13\right)^2 + 1 = \dfrac{13}{9}.$
Jadi, $\boxed{\displaystyle \lim_{x \to \infty} f(x_n) = \dfrac{13}{9}}$

[collapse]

Baca Juga: Kumpulan Soal ON MIPA-PT Matematika (Tahun 2006 – Sekarang)